LSAT and Law School Admissions Forum

Get expert LSAT preparation and law school admissions advice from PowerScore Test Preparation.

 Administrator
PowerScore Staff
  • PowerScore Staff
  • Posts: 8915
  • Joined: Feb 02, 2011
|
#27228
Complete Question Explanation

The correct answer choice is (E)

The position given in the last sentence of passage B’s second paragraph is that when copyright infringement occurs it suppresses the motivation to create new, copyrightable works, which in turn threatens the efficiency of the free-market system. To strengthen this argument you should strengthen the connection between these ideas (copyright violation :arrow: fewer copyrightable works :arrow: decreased efficiency of the free-market system). Answer choice E is correct because it strengthens the relationship between a lack of new, copyrightable works and an efficient economic system by stating that the generation of new works is a critical component of a free-market system. Answer choice A is an Opposite answer in that it weakens the author’s argument, and answer choice C, while tempting, does not strengthen the author’s position (just because the free market relies on a number of elements to operate efficiently does not mean that it relies on the creation of new, copyrightable works).
 lathlee
  • Posts: 652
  • Joined: Apr 01, 2016
|
#41087
Hi. this question killed me for one reason which i found it wording really unfair.

as in, the correct answer assumes that free market = capitalistic ?

mixed economy for example, germany and canada, also considered to be free market driven economic world?
User avatar
 Stephanie Oswalt
PowerScore Staff
  • PowerScore Staff
  • Posts: 812
  • Joined: Jan 11, 2016
|
#41182
Hi Lathlee,

I moved your question over to the thread explaining this question. Let us know if you have any additional questions! Thanks!
 jessicamorehead
  • Posts: 84
  • Joined: Jul 07, 2017
|
#44278
Hi,

I'm not sure where to find the discussion on this question. I, too, am confused on the shift in terms (free market system to capitalist). Can someone direct me there?
 Adam Tyson
PowerScore Staff
  • PowerScore Staff
  • Posts: 5153
  • Joined: Apr 14, 2011
|
#44290
Hey there lathlee and Jessica, happy to help here, although you might not like the answer. While it's true that we are not supposed to bring outside info to the table when taking this test, the authors do often make assumptions about what they consider to be common knowledge included in what someone with a college education is expected to know, and many questions will turn on those assumptions. Here, the authors of the test assume that you know that capitalism is defined, at least in part, by the working of free markets. Whether you have taken courses in economics or not, that's assumed to be common knowledge.

So what do we do when faced with something like that, when we aren't really sure about the information presented? First, we have to keep the answer as a contender. You simply cannot knock out an answer choice based on "I'm not sure" or "I'm confused". The only losers are the ones that you KNOW are incorrect, for whatever reason. Whatever is left must be a contender, and if there is only one contender then that must be the correct answer, even if we don't entirely understand it! This happens to me on a lot of science questions, by the way - I get dazed and confused by the Latin terms and technical jargon, and the next thing you know I am going "I don't understand this answer - CONTENDER!"

In this case, none of the other answers has anything to do with strengthening the relationship we were asked about, which is something like "suppression of content creation threatens the efficiency of the free market." If I want to strengthen that, I need to build a bridge between the concepts of "content creation" and "efficiency", something like "without the first thing you cannot have the second" or "if you suppress the first, you threaten the second". Answer E is the only thing that comes close, and it's not much of a stretch to equate "capitalism" and "free markets".

Sometimes, all you can do is eliminate the losers and go with whatever is left standing! Other times, you have to rely on what you think the authors think is common knowledge. Whenever I discuss this idea, I always come back in my mind to an LR question that involves plowing a field at night vs plowing it during the daytime. In that one, the authors assume that we know that the sun shines during the day and does not shine at night. Outside knowledge, to be sure, but common enough that it's fair for them to assume we know it. Capitalism involving free markets may be a little less commonly understood, but not by much among folks the authors expect to be sitting for the LSAT.

Bad news, perhaps, but I hope it helps you to start thinking more about losers, contenders, and "best answer" instead of "good answer" or "right answer" going forward!
 jessicamorehead
  • Posts: 84
  • Joined: Jul 07, 2017
|
#44305
Adam,

Thank you for your advice! I was a science major, so anything related to economics is a foreign language to me haha. But I will definitely adopt your approach of NOT knocking off something just because I don't understand it. Everything is a contender unless I can find a reason for it to be incorrect. Thank you!!
User avatar
 cd1010
  • Posts: 42
  • Joined: Jul 12, 2022
|
#105722
Hello -- I picked the correct answer, E, but I think I spent too much time on the question, because I was getting tripped up over D. Do you have a clearer answer for why D is wrong? Ultimately, I crossed it out because the argument to be strengthened is: losses threaten the efficiency of the free market system. So, D doesn't have this element (free market system). Is that right?
 Robert Carroll
PowerScore Staff
  • PowerScore Staff
  • Posts: 1787
  • Joined: Dec 06, 2013
|
#105732
cd1010,

The free-market system is the foundation of the country. Answer choice (D) claims that something else is. That's not applicable to the passage. Consider why the argument is bad in the first place, which it must be if there's a Strengthen question (perfect arguments actually can't be strengthened because they're not missing anything). The author claims that suppressing the drive of content creators will threaten the free-market system. How? What does the one have to do with the other? We could similarly say that, for instance, the use of winter chains on tires threatens the free-market system. That sounds like a wild claim, but if tire chains damage roads, then the infrastructure needed to move goods would be impaired, threatening the system. So I can say that ANYTHING threatens the free-market system, but if I do, I really owe an explanation of HOW it's threatened. The author provided no explanation. Answer choice (D) says "actually, something else is the foundation of the country." That's not addressing the gap between the two things that were already mentioned.

Robert Carroll

Get the most out of your LSAT Prep Plus subscription.

Analyze and track your performance with our Testing and Analytics Package.